Answer (D) is correct . The number of guests Kim must have to raise $10,000 can be calculated as follows: Target unit sales = (Fixed costs + Target operating income) ÷ UCM = {($150,000 + $50,000 + $4,000) + $10,000] ÷ ($500 – $12 – $8) = $214,000 ÷ $480 = 445.83
Answer (A) is incorrect because This number of guests is found by subtracting the $10,000 of desired income from the fixed costs. Answer (B) is incorrect because This number of guests is the amount needed to break even. Answer (C) is incorrect because This number of guests is found by not subtracting the variable costs per person from the revenues per person to get the contribution margin.
|